dan
Thanks Received: 155
Atticus Finch
Atticus Finch
 
Posts: 202
Joined: March 10th, 2009
 
 
 

Q9 - Although video game sales have

by dan Fri Dec 31, 1999 8:00 pm

9. (E)
Question Type: Weaken the Conclusion
What are the potential problems with the argument? In this case, the author assumes that no other part of the population will take the place of the 13 to 16 year olds when it comes to purchasing games. If we can find an answer that destroys this assumption, we’ll have our answer. Answer (E) does just that.

(A) actually strengthens the argument, the exact opposite of what we want.
(B) is out of scope (video game rentals?).
(C) is out of scope (new entertainment options?).
(D) is out of scope (different types of video games?).


#officialexplanation
 
cc4me19
Thanks Received: 0
Forum Guests
 
Posts: 7
Joined: October 25th, 2009
 
 
 

Q9 - Although video game sales have

by cc4me19 Sun Feb 07, 2010 3:52 pm

I understand that the correct answer is supposed to weaken the argument more compared with the other answer options, and to a certain degree I understand how answer choice (E) does this, when compared with the other options. What I don't understand completly is that to me (E) contradicts the argument so much so that it doesn't make sense to even be a true statement. The argument says 'over three quarters of video games sold have been purchased by people from 13 to 16 years of age', while (E) says 'Most of the people who have purchased video games over the past 3 years are over the age of 16.'
So, how can over 3/4 of video games be sold to people between 13 and 16, while at the same time most of the people who have purchased video games be over the age of 16?
User avatar
 
noah
Thanks Received: 1192
Atticus Finch
Atticus Finch
 
Posts: 1541
Joined: February 11th, 2009
 
 
 

Re: Q9 - Although video game sales have

by noah Mon Feb 08, 2010 1:29 pm

Here's a link to a full set of explanations for that test. I figured I'd point you to that before answering this; perhaps that answers it for you. If not, post a follow-up question please.

june-2007-all-sections-and-solutions-t292.html

- Noah
 
svinson.p
Thanks Received: 0
Forum Guests
 
Posts: 1
Joined: August 07th, 2013
 
 
 

Re: Q9 - Although video game sales have

by svinson.p Wed Aug 07, 2013 2:01 pm

I think the use of the word three trips people up in the stimulus. While historically, 75% of buyers have been ages 13-16, over the past THREE years most have been over age 16. Maybe the newest Halo game came out three years ago and you had to be 16 to buy it :).
User avatar
 
WaltGrace1983
Thanks Received: 207
Atticus Finch
Atticus Finch
 
Posts: 837
Joined: March 30th, 2013
 
 
trophy
Most Thanked
trophy
Most Thankful
trophy
First Responder
 

Re: Q9 - Although video game sales have

by WaltGrace1983 Thu Oct 16, 2014 7:06 pm

Historically, over 75% of the video games sold have been purchased by 13-16-year-olds
+
Number of people in this group is expected to decline steadily over the next 10 years

Video game sales will not steadily increase in the near future

I think the key thing to think about here is that we have a disconnection between what has happened "historically" and what will happen in the "near future." Historically, most people who go to law school have well-paying careers and lots of success. So is it fair to say that such a phenomenon will continue for the next three years when we all graduate from law school? NO! Over the last five or so years, the employment prospects have dramatically changed and it is not such a sure path to success anymore. Therefore, we cannot merely associate something that has happened "historically" (whatever that means) with something that will happen in the very near future. We would be much better off looking at the recent trends rather than simply the overall trends. They can provide very different analysis.

In addition, who's to say that the number of people has anything to do with how many video games are bought? Maybe the number of people will decline but - because this, albeit smaller, number of people is more obsessed with video games - the number of video games bought will increase.

(A) This either falls in line with the argument or is out of scope. Either way, we have reason to believe that - if most 17+ year olds have never purchased a video game - this makes it more likely that video game sales will not increase.

(B) Rentals has little to do with sales. However, even if we say it does, this would strengthen the argument.

(C) New entertainment options has little to do with video games sales. However, if it does, it makes it all the more likely the video game sales will drop.

(D) According to (D), there will be the same amount or more video game types. This is not exactly a strengthener but we don't really know what "video game type" does to the argument. Does the number of video game types affect the sales? We don't know.

(E) Correct. It shows recent historical trends. That is the critical notion! This is saying that, over the past 3 years, the 17+ year olds have been very involved in the video game market. Maybe the amount of sales will not decrease after all.
User avatar
 
tommywallach
Thanks Received: 468
Atticus Finch
Atticus Finch
 
Posts: 1041
Joined: August 11th, 2009
 
This post thanked 1 time.
 
 

Re: Q9 - Although video game sales have

by tommywallach Sun Oct 19, 2014 7:08 pm

No response, just saying hello. Good to see you're still on the forums, Walt! : )

-t
Tommy Wallach
Manhattan LSAT Instructor
twallach@manhattanprep.com
Image
 
asafezrati
Thanks Received: 6
Atticus Finch
Atticus Finch
 
Posts: 116
Joined: December 07th, 2014
 
 
 

Re: Q9 - Although video game sales have

by asafezrati Thu Jun 04, 2015 10:40 pm

WaltGrace1983 Wrote:(C) New entertainment options has little to do with video games sales. However, if it does, it makes it all the more likely the video game sales will drop.


I understand why E is the best answer, but how you treat C doesn't really make sense.

New entertainment options have alot to do with sales. The evolution of the gaming industry all the way to its current state is very strong proof. New technological options can increase sales. Maybe this answer choice falls because of the "can", or maybe what I know is too far from what is considered common knowledge by the LSAT.